Difference between revisions of "2021 AMC 10A Problems/Problem 19"
(→Solution) |
(→Solution 1) |
||
Line 5: | Line 5: | ||
== Solution 1 == | == Solution 1 == | ||
+ | This is what the diagram looks like: | ||
<asy> | <asy> | ||
size(10cm); | size(10cm); | ||
Line 13: | Line 14: | ||
filldraw((-3,3)--(3,3)--(3,-3)--(-3,-3)--cycle, grey); | filldraw((-3,3)--(3,3)--(3,-3)--(-3,-3)--cycle, grey); | ||
</asy> | </asy> | ||
+ | Now, the area of the shaded region is just a square with side length <math>6</math> with four semicircles of radius <math>3</math>. | ||
+ | The area is <math>6\cdot6+4\cdot \frac{9\pi}{2} = 36+18\pi</math>. The answer is <math>36+18</math> which is <math>\boxed{\textbf{(E) }54}</math> | ||
+ | ~ Bryguy | ||
Revision as of 17:10, 11 February 2021
Problem 19
The area of the region bounded by the graph ofis , where and are integers. What is ?
Solution 1
This is what the diagram looks like: Now, the area of the shaded region is just a square with side length with four semicircles of radius . The area is . The answer is which is ~ Bryguy
https://artofproblemsolving.com/wiki/index.php/File:Image_2021-02-11_111327.png (someone please help link file thanks)
Video Solution (Using absolute value properties to graph)
~ pi_is_3.14